You are on page 1of 66

TEST - 21

User Name :

chandan paswan

Total Marks :

200

Mark Scored :

67.33

IA
S

1 Consider the following statements.


1. Assertion (A): Africa does not have coal reserves.
2. Reason (R): African landscape is composed entirely of sedimentary and crystalline
rocks.
In the context of the above, which of these is correct?
A. A is correct, and R is an appropriate explanation of A.
B. A is correct, but R is not an appropriate explanation of A.
C. A is correct, but R is incorrect
D. Both A and R are incorrect
User Answer : B
Correct Answer : D
Answer Justification :

H
TS

Justification: South Africa and Zimbabwe are the only two African countries
recognised by the IEA as having proven coal reserves.
Nigeria is also believed to have abundant coal reserves left unexploited
Entire African landscape is not covered with sedimentary or crystalline rocks. The
presence of many volcanoes clearly indicates that there are igneous rocks.
Moreover, oil reserves are plenty in Africa. So, R is incorrect.

Q Source: Revision: 8th NCERT: Geography

IN

SI

2 The Government of India (GoI) Act of 1935 provided for a three-fold emumenration,
viz., federal, provincial and concurrent. The present Constitution follows the scheme of this
act but with some difference(s), which are
1. In the GoI Act 1935, residuary powers were given to the governor-general of India,
which now vests with the Centre
2. In the GoI Act 1935, Centre had no right to legislate in the concurrent list without
the consent from the concerned state.
Which of the above is/are correct?
A. 1 only
B. 2 only
C. Both 1 and 2
D. None
User Answer : A
Correct Answer : A
Answer Justification :
Justification: The present Constitution follows the scheme of this act but with one

(C) Insights Active Learning. | All rights reserved.

www.insightsias.com

TEST - 21

User Name :

chandan paswan

Total Marks

200

Mark Scored

67.33

difference, that is, under this 1935 act, the residuary powers were given neither to
the federal legislature nor to the provincial legislature but to the governor-general of
India.
As of now, it vests with the Centre.

IA
S

The provision with regard to concurrent list was same as earlier. So, 2 is incorrect.
Q Source: Chapter on Parliament: Indian Polity: M laxmikanth

SI

H
TS

3 Once a language is declared as a classical language by the Government of India (GoI),


what are the advantages conferred to it?
1. It must be taught in all Centre-funded Universities
2. The speakers of the language are given preference in government jobs
3. The GoI gives financial assistance for setting up a centre of excellence for the study
of that language.
4. The GoI sets up dedicated missions to promote that language abroad
Select the correct answer using the codes below
A. 1 and 2 only
B. 3 only
C. 1, 2 and 4 only
D. 3 and 4 only
User Answer : B
Correct Answer : B
Answer Justification :

IN

Justification & Learning: The criteria for declaring a language as classical


mandates high antiquity of its early texts/recorded history over a period of
1,500-2,000 years, a body of ancient literature/texts which is considered a valuable
heritage by generations of speakers and a literary tradition that is original and not
borrowed from another speech community.
Once a language is declared classical, it gets financial assistance for setting
up a centre of excellence for the study of that language and also opens up an
avenue for two major awards for scholars of eminence.
Besides, the University Grants Commission can be requested to create to
begin with at least in Central Universities a certain number of professional
chairs for classical languages for scholars of eminence in the language.

Q Source: http://www.ccrtindia.gov.in/literaryarts.php\

(C) Insights Active Learning. | All rights reserved.

www.insightsias.com

TEST - 21

User Name :

chandan paswan

Total Marks :

200

Mark Scored :

67.33

IA
S

4 The characteristic feature of Africa's latitudinal extent is that it extends


A. Over tropical, temperate as well as frigid zones.
B. Both Northern and Southern hemispheres almost equally
C. Entirely to Southern hemisphere
D. Entirely to Northern hemisphere
User Answer : B
Correct Answer : B
Answer Justification :

IN

SI

H
TS

Justification: The image below verifies option (a).

Q Source: Revision previous tests syllabus: World Geography: 10th NCERT


5 Consider the following statements.
1. Assertion (A): In the North Indian Ocean, there is a complete reversal of the

(C) Insights Active Learning. | All rights reserved.

www.insightsias.com

TEST - 21

User Name :

chandan paswan

Total Marks

200

Mark Scored

67.33

IA
S

direction of ocean currents between summer and winter.


2. Reason (R): Monsoon winds change direction from South-west to North-east
In the context of the above, which of these is correct?
A. A is correct, and R is an appropriate explanation of A.
B. A is correct, but R is not an appropriate explanation of A.
C. A is incorrect, but R is incorrect
D. Both A and R are incorrect
User Answer :
Correct Answer : A
Answer Justification :

Justification: The currents of the Indian Ocean are modified by the landmass in the
north and influenced by the Monsoon winds.

H
TS

In the North Indian Ocean, a complete reversal of direction of Monsoon Current,


twice a year is observed between summer and winter because the Monsoon winds
change their direction according to seasons.

South-West Monsoon Current(warm) o In the summer season, the monsoon


winds blow from south west to north-east. Consequently, the ocean current
also flows from south-west to north-east direction. o The main current moves
in the Arabian Sea and the Bay of Bengal along the coast of India.
North-East Monsoon Current (warm) o In the winter season, the monsoon
winds blow from north-east to south-west. The ocean current also flows from
north-east to south-west under the influence of these winds.

SI

Q Source:Chapter 12: Goh Cheng Leong: Certificate Physical and Human


Geography

IN

6 The Election Commission registers political parties for the purpose of elections and
grants them recognition as national or state parties on the basis of their
A. Membership size
B. Regional Presence
C. Poll performance
D. Party exchequer
User Answer : C
Correct Answer : C
Answer Justification :
Learning: The other parties that are not recognized by the ECI are simply declared
as registered-unrecognised parties.

(C) Insights Active Learning. | All rights reserved.

www.insightsias.com

TEST - 21

User Name :

chandan paswan

Total Marks :

200

Mark Scored :

67.33

The recognition granted by the Commission to the parties determines their right to
certain privileges like allocation of the party symbols, provision of time for political
broadcasts on the state-owned television and radio stations and access to electoral
rolls.

IA
S

Q Source: Chapter 65: Indian Polity: M Laxmikanth

H
TS

7 Carbon monoxide (CO) is one of the most serious air pollutants. It is released in air by
1. Automobile exhaust
2. Volcanic activity
3. Forest and bushfires
Select the correct answer using the codes below.
A. 1 and 2 only
B. 2 and 3 only
C. 1 and 3 only
D. 1, 2 and 3
User Answer : D
Correct Answer : D
Answer Justification :

Learning: It is produced as a result of incomplete combustion of carbon. Carbon


monoxide is mainly released into the air by automobile exhaust. Other sources,
which produce CO, involve incomplete combustion of coal, firewood, petrol, etc.

SI

Carbon monoxide is present in small amounts in the atmosphere, chiefly as a


product of volcanic activity but also from natural and man-made fires (such as forest
and bushfires, burning of crop residues, and sugarcane fire-cleaning).

IN

Q Source: Revision: 11th NCERT: Chemistry

8 Estuaries are one of the most biologically productive regions in the World because
A. It receives high amount of nutrients from both fresh and marine water.
B. It witnesses high wave action
C. It is not subject to tidal fluctuations unlike the open sea
D. Warm and cold currents meet at estuaries.
User Answer : A
Correct Answer : A
Answer Justification :
Justification & Learning: Estuaries are unique places, strongly affected by tidal

(C) Insights Active Learning. | All rights reserved.

www.insightsias.com

TEST - 21

User Name :

chandan paswan

Total Marks

200

Mark Scored

67.33

action, where land and river and sea merge into a dynamic natural complex. But,
there is little wave action unlike the open sea. So, (b) and (c) are incorrect.
Estuaries rank along with tropical rainforests and coral reefs as the world's most
productive ecosystems, more productive than both the rivers and the ocean that
influence them from either side.

IA
S

In an estuary, nutrient-rich river waters combine with warmer, light infused


shallow coastal waters and the upwelling of nutrient-rich deep ocean waters
to generate primary productivity
The mixing of lighter fresh water and heavier salt water trap and circulate
nutrients such that they are often retained and recycled by benthic (bottom
dwelling) organisms to create a self-enriching system.

H
TS

Q Source: 9th Standard ICSE Textbook: Unit I

IN

SI

9 Andhra Pradesh Agriculture Project 'Harita-Priya' has won the prestigious World Summit
on the Information Society (WSIS) 2016 prize in the e-Agriculture category. The HaritaPriya Project
A. Uses wireless sensor networks to collect micro-climatic information from
farms and issues alert to farmers
B. Connects all agriculture mandis through an internet network
C. Issues regular advisory to farmers on best financial practices to tackle farmer
suicides
D. Creates a database of all farmers in the state to ease disbursement of crop
failure insurance
User Answer : A
Correct Answer : A
Answer Justification :
Learning: Harita-Priya (Harmonized Information of Agriculture, Revenue and
Irrigation for a Transformation Agenda Precision Technology for Agriculture)
Project is meant to collect very specific climate data from farms and give farmers
personalised information through SMS.
The data is used to create advisories and alerts on irrigation schedules and pests and
plant diseases
The Centre for Development of Advanced Computing (CDAC) in Hyderabad has
developed the project and is having a pilot run in Anantpur district.

(C) Insights Active Learning. | All rights reserved.

www.insightsias.com

TEST - 21

User Name :

chandan paswan

Total Marks :

200

Mark Scored :

67.33

Q Source:
http://www.thehindu.com/todays-paper/tp-in-school/international-recognition-for-an
dhra-agriculture-project/article8557566.ece

H
TS

IA
S

10 What is/are the difference(s) between troposphere and stratosphere?


1. Jet aircrafts fly in stratosphere, not in troposphere
2. Stratosphere is the layer with auroras, not troposphere
3. Weather patterns occur in the troposphere, not in stratosphere
Select the correct answer using the codes below.
A. 1 and 2 only
B. 2 and 3 only
C. 1 and 3 only
D. 1, 2 and 3
User Answer : B
Correct Answer : C
Answer Justification :

Justification: Statement 1: Many jet aircrafts fly in the stratosphere because it is


very stable.
Statement 2: It is thermosphere, since it contains charged particles.

SI

Statement 3: Weather patterns occur in the troposphere because the air near the
ground is warmer than the air at higher altitudes; this phenomenon is a result of the
fact that the ground absorbs and radiates heat from the sun
Q Source: Revision previous tests syllabus: 9th NCERT: Geography

IN

11 Art historians had to acquire familiarity with hagiographies of the Buddha in order to
understand Buddhist sculpture. What does the frequently used wheel in the hagiographies
stand for?
A. Meditation of the Buddha
B. The Stupa
C. First sermon of the Buddha
D. Previous births of Buddha
User Answer : C
Correct Answer : C
Answer Justification :
Justification: According to hagiographies, the Buddha attained enlightenment while

(C) Insights Active Learning. | All rights reserved.

www.insightsias.com

TEST - 21

User Name :

chandan paswan

Total Marks

200

Mark Scored

67.33

meditating under a tree.


The empty seat was meant to indicate the meditation of the Buddha, and the stupa
was meant to represent the mahaparinibbana.

IA
S

The wheel stood for the first sermon of the Buddha, delivered at Sarnath because it
was here that he set the wheel of Dhamma in motion
Q Source: Revision past tests syllabus: 11th TamilNadu History Textbook

H
TS

12 Which of the following schedules of the constitution gives a clear constitutional


recognition to the existence of the system of "political parties" in India?
A. Seventh Schedule
B. Tenth Schedule
C. Third Schedule
D. Ninth Schedule
User Answer : B
Correct Answer : B
Answer Justification :

Justification & Learning: The Tenth Schedule of the Constitution (which


embodies the anti-defection law) is designed to prevent the evil or mischief of
political defections motivated by the lure of office or material benefits or other
similar considerations

SI

Since the anti-defection provisions clearly mention the existence of political parties
(and members being disqualified on leaving its membership), the Tenth schedule
recognizes the system of political parties.

IN

Q Source: Chapter 67: Indian Polity: M Laxmikanth

13 Fly ash is one of the coal combustion products. Use of fly ash in agriculture can
1. Increase crop yield
2. Increase concentrations of arsenic in crops
3. Enhances water holding capacity of land
Select the correct answer using the codes below.
A. 2 only
B. 1 and 3 only
C. 2 and 3 only
D. 1, 2 and 3

(C) Insights Active Learning. | All rights reserved.

www.insightsias.com

TEST - 21

User Name :

chandan paswan

Total Marks :

200

Mark Scored :

67.33

User Answer :
Correct Answer : D
Answer Justification :

IA
S

Justification: Statement 1 and 3: Fly ash can be used as a soil ameliorate that may
improve physical, chemical and biological properties of the degraded soils and is a
source of readily available plant micro-and macro-nutrients. The high concentration
of elements (K, Na, Zn, Ca, Mg and Fe) in fly-ash increases the yield of many
agricultural crops.
Statement 2: However, fly ash contains various amounts of toxic metals. And
studies have shown that food crops grown in large amounts can soak up hazardous
concentrations of arsenic.

H
TS

Q Source: 9th Standard ICSE Textbook: Unit II

14 India followed the policy of Non-Alignment post-independence in response to


A. Economic backwardness of India
B. Impending wars with South Asian neighbours
C. Cold war rivalry
D. Former agreements between South Asian neighbours
User Answer : C
Correct Answer : C
Answer Justification :

SI

Justification: A case can be made for options (a) and (b), but these are not as strong
as option (c), which is the most appropriate option.

IN

Option (a) because, policy of non-alignment helped us maintain an independent


foreign policy and stop neo-colonialism, which could have again sapped Indias
resouces
Option (b) because joining a cold war alliance may have further aggravated rivalry
with neighbours
But, (c) is most appropriate as it was the cold war and in this context, Jawaharlal
Nehru observed: We propose to keep away from the power politics of groups,
aligned against one another, which have led in the past to world wars and which
may again lead to disasters on an even vaster scale.
Q Source: Chapter 70: Indian Polity: M Laxmikanth

(C) Insights Active Learning. | All rights reserved.

www.insightsias.com

TEST - 21

User Name :

chandan paswan

Total Marks

200

Mark Scored

67.33

IA
S

15 Which of the following statements about Bitcoins is INCORRECT?


A. It can be traded on an open market.
B. It is a peer-to-peer digital currency
C. It does not require any intermediary like a bank for transaction
D. It can only be used for speculation and not for any purchase
User Answer : D
Correct Answer : D
Answer Justification :

Justification: Bitcoin is a virtual currency that is unregulated by any central bank or


government, but still works for purchasing goods and services from retailers willing
to accept it. So, (d) is correct

H
TS

It can also be traded on an open market and its exchange rate fluctuates much like a
stock market i.e. based on the demand.
Bitcoin is a distributed peer-to-peer digital currency that functions without the intermediation of any central authority
Bitcoin is also called a cryptocurrency since it is decentralized and uses
cryptography to prevent double-spending, a significant challenge inherent to digital
currencies.

SI

Q Source:
http://www.thehindu.com/business/all-you-need-to-know-about-bitcoin/article85474
40.ece

IN

16 In 1949, India declared the continuation of her full membership of the Commonwealth
of the Nations. But, this extra-constitutional declaration did not affect India's sovereignty in
any manner as
1. The position of Head of Commonwealth rotates among member nations on a regular
basis.
2. The Commonwealth is a voluntary association of independent nations
Which of the above is/are correct?
A. 1 only
B. 2 only
C. Both 1 and 2
D. None
User Answer : C
Correct Answer : B
Answer Justification :

(C) Insights Active Learning. | All rights reserved.

www.insightsias.com

10

TEST - 21

User Name :

chandan paswan

Total Marks :

200

Mark Scored :

67.33

Justification: Statement 1: Under the formula of the London Declaration, Queen


Elizabeth II is the Head of the Commonwealth. So, 1 is wrong.

IA
S

Statement 2: Being a member of commonwealth also does not affect Indias


republican character as India neither pays final allegiance to the British Crown nor
the latter has any functions to discharge in relation to India since it is a voluntary
organization.
Q Source: Chapter 70: Indian Polity: M Laxmikanth

H
TS

17 Generally fogs are more common over sea than land. Which of the following can be the
reason(s)?
1. Ocean salt provides condensation nuclei for the formation of fog.
2. Moist air blown by wind over a cool surface will form fog.
Which of the above is/are correct?
A. 1 only
B. 2 only
C. Both 1 and 2
D. None
User Answer : B
Correct Answer : C
Answer Justification :

SI

Justification: Statement 1: Sea fog forms when the condensation nuclei is salt. Salt
is, of course, very common near the ocean where it is kicked up into the atmosphere
by the breaking waves. Salt is a unique condensation nuclei in that it will allow fog
to form when the humidity is as low as 70%.

IN

Statement 2: When wind blows moist air over a cool surface the air will cool and
advection fog will form. Advection fog is very common at sea when tropical winds
pass over cooler waters and on land when a warm front passes over heavy snow.
Q Source: Chapter 13: Goh Cheng Leong: Certificate Physical and Human
Geography

18 India has not signed the Nuclear Non-proliferation Treaty (NPT) of 1968 and the
Comprehensive Test Ban Treaty (CTBT) of 1996 because
A. India did not have nuclear weapons when the treaties were signed.
B. The treaties were discriminatory and hegemonistic in nature in India's view
C. The treaty demands sharing of crucial and strategic information of a nation's

(C) Insights Active Learning. | All rights reserved.

www.insightsias.com

11

TEST - 21

User Name :

chandan paswan

Total Marks

200

Mark Scored

67.33

IA
S

nuclear facilities.
D. The treaty demands complete control of the concerned Nuclear authority of
the nation
User Answer :
Correct Answer : B
Answer Justification :
Justification: By not signing the Nuclear Non-proliferation Treaty (NPT) of 1968
and the Comprehensive Test Ban Treaty (CTBT) of 1996, India has kept its nuclear
options open
India opposses NPT and CTBT due to their discriminatory and hegemonistic nature.
They perpetuate an international system in which only five nations (USA, Russia,
China, UK and France) can legitimately possess nuclear weapons.

H
TS

India is already sharing information regarding its nuclear facilities with IAEA. So,
this could not have been a contention while signing the NPT and CTBT. (c) will be
wrong thus.
Q Source: Chapter 70: Indian Polity: M Laxmikanth

IN

SI

19 Which of the following pollutants are emitted by Iron and Steel Industry?
1. Mercury
2. Sulphides
3. Fluorine
4. Chlorides
5. Cadmium
Select the correct answer using the codes below
A. 1, 2 and 5 only
B. 3 and 4 only
C. 2 and 5 only
D. 1, 2, 3, 4 and 5
User Answer :
Correct Answer : A
Answer Justification :
Justification: Statement 1: Mercury is emitted in air from the furnaces used in steel
making.
Statement 2: Iron ore is also found in sulphide form in the crust.

(C) Insights Active Learning. | All rights reserved.

www.insightsias.com

12

TEST - 21

User Name :

chandan paswan

Total Marks :

200

Mark Scored :

67.33

Statement 3 and 4: Fluorine and chlorides are emitted in the chemical industry
Statement 5: Cadmium enters the industrial economy inadvertently as a trace
impurity of high-volume raw materials. The most important of these are phosphate
ores, coal, oil, and iron ore.

IA
S

Q Source: 9th Standard ICSE Textbook: Unit II

H
TS

20 Which of these is/are a part of the capital receipts of the Government of India?
1. Borrowings from the Reserve Bank of India (RBI)
2. Money accrued from sale of government bonds to the public
Which of the above is/are correct?
A. 1 only
B. 2 only
C. Both 1 and 2
D. None
User Answer : C
Correct Answer : C
Answer Justification :

Justification: A receipt is a capital receipt if it satisfies any one of the two


conditions:

SI

(i) The receipts must create a liability for the government. For example, Borrowings
are capital receipts as they lead to an increase in the liability of the government.
However, tax received is not a capital receipt as it does not result in creation of any
liability.

IN

(ii) The receipts must cause a decrease in the assets. For example, receipts from sale
of shares of public enterprise is a capital receipt as it leads to reduction in assets of
the government
Since, both borrowing and sale of government bonds create liability, they are called
capital receipts.
Q Source: Chapter on Public Finance: 12th Macroeconomics NCERT

21 Consider the following statements


1. Assertion (A): There is a dedicated Ministry for Agriculture and related affairs at the
Central level.

(C) Insights Active Learning. | All rights reserved.

www.insightsias.com

13

TEST - 21

User Name :

chandan paswan

Total Marks

200

Mark Scored

67.33

IA
S

2. Reason (R): Agriculture, including agricultural education and research fall under the
Union List in the Seventh Schedule of the Constitution.
In the context of the above, which of these is correct?
A. A is correct, and R is an appropriate explanation of A
B. A is correct, but R is not an appropriate explanation of A
C. A is correct, but R is incorrect
D. A is incorrect, but R is correct
User Answer :
Correct Answer : C
Answer Justification :
Justification: Agriculture, including agricultural education and research falls under
the State list.

H
TS

The Ministry at the Centre has been formed due to a variety of reasons (only some
are listed below):

SI

Agriculture is a strategic sector, and is vital for the livelihood for a majority
of Indian population.
It is essential to coordinate and assist the activities of the states to achieve
synergy and higher agriculture growth.
Floods, droughts, other disasters that occur on regional scale or national scale
affect agriculture. Centres intervention is important to stop this.
The centre needs to work for the welfare the farmer community which is so
closely associated with Agriculture.
States may not be financially and technically capable to support and improve
agriculture in the state.

IN

Q Source: Appendix II: Indian Polity: M Laxmikanth

22 The Table of Precedence is related to the rank and order of the officials of the Union
and State Governments. Consider the following
1. Cabinet Ministers of the Union
2. Former presidents
3. Governors of states within their respective states
4. Vice-President
5. Chief Justice of India
6. Prime Minister
Which of the following codes show the correct order of precedence?
A. 6>4>2>1>5>3

(C) Insights Active Learning. | All rights reserved.

www.insightsias.com

14

TEST - 21

User Name :

chandan paswan

Total Marks :

200

Mark Scored :

67.33

Justification & Learning: It goes like this:

IA
S

B. 4>2>6>3>1>5
C. 4>6>3>2>5>1
D. 6>2>4>5>3>1
User Answer :
Correct Answer : C
Answer Justification :

H
TS

President
Vice-President
Prime Minister
Governors of states within their respective states
Former presidents; Deputy Prime Minister
Chief Justice of India; Speaker of Lok Sabha
Cabinet Ministers of the Union; Chief Ministers of States within their
respective States; Deputy Chairman, Planning Commission; Former Prime
Ministers; Leaders of Opposition in Rajya Sabha and Lok Sabha
Q Source: Appendix III: Indian Polity: M Laxmikanth

IN

SI

23 Neem-coating of Urea is an important recent reform in the fertiliser sector. This has
been done to
A. Improve the shell life of Urea
B. Reduce the toxic effects of the urea on crops
C. Reduce the cost of nitrogen-based fertilizers
D. Prevent diversion of urea to industrial uses
User Answer : B
Correct Answer : D
Answer Justification :
Justification & Learning: Urea is an important supplier of nitrogen, which is
necessary for the development of plants. But only 30-40 per cent of nitrogen present
in the urea is utilised by crops
When farmers use conventional urea, about half the applied nitrogen is not
assimilated by the plant and leaches into the soil, causing extensive groundwater
contamination
Spraying urea with neem oil slows the release of nitrogen, by about 10 to 15 per
cent, concomitantly reducing consumption of the fertiliser.

(C) Insights Active Learning. | All rights reserved.

www.insightsias.com

15

TEST - 21

User Name :

chandan paswan

Total Marks

200

Mark Scored

67.33

The neem-coating also precludes an age-old malpractice of this cheap fertiliser


being diverted for use in the chemical industry and, most harmfully in states like
Punjab and Haryana, as an additive in milk to whiten it
Q Source: Chapter 9: Economic Survey 2015-16: Volume I

H
TS

IA
S

24 Ujwal DISCOM Assurance Yojana (UDAY) envisages


1. Centre taking over all the debts of electricity DISCOMs
2. Tiered electricity tariffs for different sections of Indian population
3. Increased supply of domestic coal to substitute for imported coal.
Select the correct answer using the codes below
A. 1 and 2 only
B. 2 and 3 only
C. 3 only
D. 1, 2 and 3
User Answer : A
Correct Answer : C
Answer Justification :

Justification: Statement 1: States shall take over 75 per cent of discom debt
outstanding as of September 2015. States shall take over future losses of discoms in
a phased manner

SI

Statement 2: No such provision. But, to maintain tarrifs at sustainable levels, there is


provision or reduction of Aggregate Technical & Commercial (AT&C) losses to 15
per cent by 2018-19, and reduction in difference between average cost of supply and
average revenue realized (ARR) by 2018-19

IN

Statement 3: This is to reduce the cost of generating electricity as well as reduce


dependence on imports.
Q Source: Chapter 10: Economic Survey 2015-16: Volume I

25 Which of the following may be categorized as primary producers?


1. Bacteria
2. Algae
3. Lichens
Select the correct answer using the codes below.
A. 1 and 2 only
B. 1 and 3 only

(C) Insights Active Learning. | All rights reserved.

www.insightsias.com

16

TEST - 21

User Name :

chandan paswan

Total Marks :

200

Mark Scored :

67.33

C. 2 and 3 only
D. 1, 2 and 3
User Answer : D
Correct Answer : D
Answer Justification :

IA
S

Learning: Primary producers, also called autotrophs, are organisms that can
produce their own food.
In almost all cases these are photosynthetically active organisms.

Fungi and other organisms that gain their biomass from oxidizing organic materials
are called decomposers and are not primary producers.

H
TS

However, lichens located in tundra climates are an exceptional example of a primary


producer that, by mutualistic symbiosis, combine photosynthesis by algae (or
additionally nitrogen fixation by cyanobacteria) with the protection of a decomposer
fungus
Q Source: 9th Standard ICSE Textbook: Unit I

IN

SI

26 Consider the following about Asteroids


1. The move around the Sun
2. They can be found between the orbits of Mars and Jupiter
3. Asteroids are parts of Sun that separated many years back
Select the correct answer using the codes below
A. 1 and 2 only
B. 1 and 3 only
C. 2 and 3 only
D. 1, 2 and 3
User Answer : D
Correct Answer : A
Answer Justification :
Justification: Apart from the stars, planets and satellites, there are numerous tiny
bodies which also move around the sun. These bodies are called asteroids.
They are found between the orbits of Mars and Jupiter. So, 1 and 2 are correct
Scientists are of the view that asteroids are parts of a planet which exploded many
years back. So, 3 is wrong.

(C) Insights Active Learning. | All rights reserved.

www.insightsias.com

17

TEST - 21

User Name :

chandan paswan

Total Marks

200

Mark Scored

67.33

Meteoroids also move around the sun. The largest asteroid is the Ceres
Q Source: Revision: 6th NCERT: Geography

H
TS

IA
S

27 These cover only about 7% of earth's surface but are home to 40% of World's plants and
animal species. There can be found almost constant year-round humidity. Multiple storey
of broad-leafed trees can be found. Which of the following is most similar to this
description?
A. Tropical Deciduous forests
B. Temperate Deciduous forests
C. Tropical rainforests
D. Mediterranean vegetation
User Answer :
Correct Answer : C
Answer Justification :
Learning: A rainforest has an almost constant year-round humidity. This is because
a rainforest receives lots of rain and has a constantly warm temperature.

As a result of the consistent temperatures and humidity and the constant yearly
rainfall, a rainforest only has two seasons. Tropical rainforests have a wet season
and a dry season

SI

A tropical rainforest has four distinct layers: the emergent layer (tallest trees), the
canopy, the understorey and the forest floor.
Q Source: 9th Standard ICSE Textbook: Unit I

IN

28 As the office of the Speaker is vested with great prestige, position and authority,
independence and impartiality becomes its sine qua non. Thus, she is provided with a
security of tenure. She can be removed only by
A. An independent inquiry conducted by the Supreme Court
B. A resolution passed by the Lok Sabha by a special majority
C. The President on an independent inquiry conducted by a Committee of Lok
Sabha
D. None of the above
User Answer : B
Correct Answer : D
Answer Justification :

(C) Insights Active Learning. | All rights reserved.

www.insightsias.com

18

TEST - 21

User Name :

chandan paswan

Total Marks :

200

Mark Scored :

67.33

Justification: She can be removed only by a resolution passed by the Lok Sabha by
an absolute majority (ie, a majority of the total members of the House) and not by an
ordinary majority (ie, a majority of the members present and voting in the House) or
special majority (2/3rd).

IA
S

This motion of removal can be considered and discussed only when it has the
support of at least 50 members.
Q Source: Chapter on Parliament: Indian Polity: M laxmikanth

H
TS

29 Tribal rights in India are granted and protected by


1. Fundamental Rights
2. Forest Rights Act
3. The Panchayats (Extension to the Scheduled Areas) Act (PESA)
4. Fifth and Sixth Schedule of the Constitution
Select the correct answer using the codes below.
A. 1, 3 and 4 only
B. 2 and 3 only
C. 1 and 4 only
D. 1, 2, 3 and 4
User Answer : D
Correct Answer : D
Answer Justification :

SI

Justification: Statement 1: Article 14, 15, 16, 19, 25-30 protect tribal rights.
Statement 2: It allows tribals to use minor forest produce and safeguard forest
resources.

IN

Statement 3: The concerned rights have been covered in last few tests
Statement 4: Autonomous regions have been covered in sixth schedule. Fifth
schedule has special provisions for tribal administration
Q Source: Chapter on rights of vulnerable sections: Indian Polity: M Laxmikanth

30 Drain of wealth from India occurred due to which of the following?


1. Plunder and taxation
2. Employment of Englishmen in India
3. Investment of British capital in India

(C) Insights Active Learning. | All rights reserved.

www.insightsias.com

19

4. Free and unequal trade


Select the correct answer using the codes below
A. 1, 2 and 4 only
B. 3 and 4 only
C. 2 and 3 only
D. 1, 2, 3 and 4
User Answer : D
Correct Answer : D
Answer Justification :

chandan paswan

Total Marks

200

Mark Scored

67.33

IA
S

TEST - 21

User Name :

Justification: Statement 1: Taxation that was received was not returned to the
Indians in form of welfare transfers, government spending. Instead it was used to
further the agenda of the British empire.

H
TS

Statement 2: Their salaries were paid from Indian wealth and were even sent abroad.
Statement 3: Many entrepreneurs were guaranteed returns on their investments for
e.g. in railways, and took away the investment as well as the profits back to
England.
Q Source: Revision: Indias struggle for Independence: Bipin Chandra

IN

SI

31 A parliamentary committee means a committee that


1. Is appointed or elected by the House or nominated by the Speaker / Chairman
2. Presents its report to the House or to the Speaker / Chairman
3. Has a secretariat provided by the Lok Sabha / Rajya Sabha
Select the correct answer using the codes below.
A. 1 and 2 only
B. 1 and 3 only
C. 2 and 3 only
D. 1, 2 and 3
User Answer : D
Correct Answer : D
Answer Justification :
Justification:It also works under the direction of the Speaker / Chairman of the
House.
Any committee consisting of MPs will not necessarily be a Parliamentary
Committee.

(C) Insights Active Learning. | All rights reserved.

www.insightsias.com

20

TEST - 21

User Name :

chandan paswan

Total Marks :

200

Mark Scored :

67.33

For e.g. the consultative committees, which also consist of members of Parliament,
are not parliamentary Committees as they do not fulfill above four conditions.
Q Source: Chapter on Parliament: Indian Polity: M laxmikanth

H
TS

IA
S

32 Consider the following statements.Assertion (A): The constitution doesn't recognize any
tribal language.Reason (R): The constitution recognizes only classical languages. In the
context of the above, which of these is correct?
A. A is correct, and R is an appropriate explanation of A.
B. A is correct, but R is not an appropriate explanation of A.
C. A is correct, but R is incorrect.
D. Both A and R are incorrect.
User Answer :
Correct Answer : D
Answer Justification :
Justification:The eighth schedule of the Constitution recognizes about 22
languages.
Two tribal languages, Bodo and Santhali are recognised by the Constitution.

Sanskrit, Tamil, Telugu, Kannada, Odiya and Malayalam are the classical languages
as per the status accorded by the Government of India.

SI

The constitution, thus, recognizes both classical and non-classical languages. Both A
and R are incorrect.
Q Source:http://www.ccrtindia.gov.in/literaryarts.php\

IN

"

33 'Red Tide' in water bodies occurs due to


A. Elimination of aquatic population living in inter-tidal zones
B. Algal blooms
C. Irregular tidal made coastal features
D. Rising of unusually high tides in backwaters
User Answer : B
Correct Answer : B
Answer Justification :
Learning: Red tide is a phenomenon caused by algal blooms (Wikipedia definition)

(C) Insights Active Learning. | All rights reserved.

www.insightsias.com

21

TEST - 21

User Name :

chandan paswan

Total Marks

200

Mark Scored

67.33

during which algae become so numerous that they discolor coastal waters (hence the
name "red tide").
The algal bloom may also deplete oxygen in the waters and/or release toxins that
may cause illness in humans and other animals.

Q Source: 9th Standard ICSE Textbook: Unit I

IA
S

Major factors influencing red tide events include warm ocean surface temperatures,
low salinity, high nutrient content, calm seas, and rain followed by sunny days
during the summer months.

SI

H
TS

34 Wetlands are of immense ecological importance. They perform which of the following
functions?
1. Water purification
2. Buffering shorelines against erosion
3. Mitigating floods
4. Recycling of nutrients
Select the correct answer using the codes below.
A. 1, 2 and 3 only
B. 1 and 4 only
C. 2 and 3 only
D. 1, 2, 3 and 4
User Answer : A
Correct Answer : D
Answer Justification :

IN

Justification: Wetlands are among the most productive ecosystems in the world,
comparable to rain forests and coral reefs.
The combination of shallow water, high levels of nutrients and primary productivity
is ideal for the development of organisms that form the base of the food web.
This link explains the functions very well in detail.
http://www.personal.ceu.hu/students/03/nature_conservation/wwddetail/Funct_serv.
html
Q Source:9th Standard ICSE Textbook: Unit I

(C) Insights Active Learning. | All rights reserved.

www.insightsias.com

22

TEST - 21

User Name :

chandan paswan

Total Marks :

200

Mark Scored :

67.33

IA
S

35 In case of General elections, the Model Code of Conduct for guidance of candidates and
political parties comes immediately into effect after
A. Announcement of the schedule of elections
B. Dissolution of Lok Sabha
C. Nomination filing of candidates has been over
D. The President orders for the same
User Answer : C
Correct Answer : A
Answer Justification :
Learning: The Commission normally announces the schedule of elections in a
major press conference a few weeks before the formal process is set in motion.

H
TS

The Model Code of Conduct comes immediately into effect after such
announcement.

The formal process for the elections starts with the Notification or Notifications
calling upon the electorate to elect Members of a House.
Q Source: Chapter 65: Indian Polity: M Laxmikanth

IN

SI

36 Consider the following statements about microscopic organism that lives on corals.
1. They assist the coral in nutrient production through its photosynthetic activities.
2. They impart coloration to coral reefs.
3. They are expelled in case of high stress to corals.
The above refers to?
A. Phytoplankton
B. Cyanobacteria
C. Zooxanthellae
D. None of the above
User Answer : A
Correct Answer : C
Answer Justification :
Justification:Most reef-building corals contain photosynthetic algae, called
zooxanthellae, that live in their tissues. The corals and algae have a mutualistic
relationship.
The coral provides the algae with a protected environment and compounds they
need for photosynthesis. In return, the algae produce oxygen and help the coral to
remove wastes.

(C) Insights Active Learning. | All rights reserved.

www.insightsias.com

23

TEST - 21

User Name :

chandan paswan

Total Marks

200

Mark Scored

67.33

In addition to providing corals with essential nutrients, zooxanthellae are responsible


for the unique and beautiful colors of many stony corals.

Q Source:9th Standard ICSE Textbook: Unit I

IA
S

Sometimes when corals become physically stressed, the polyps expel their algal
cells and the colony takes on a stark white appearance. This is commonly described
as "coral bleaching"

H
TS

37 The Supreme Court has recently struck down the TRAI Regulation which made it
mandatory for telecom companies to compensate subscribers for call drops. The TRAI
order was nullified because
A. Call drops do not violate consumer's freedom of expression
B. TRAI cannot financially punish telecom companies
C. It was against the Rule of law as enshrined in Article 14 of the constitution
D. The SC found it to be unreasonable and arbitrary
User Answer :
Correct Answer : D
Answer Justification :

Justification: SC noted that the TRAI had no empirical evidence to corroborate that
call drops were attributable to deficiency in services by the service providers.

SI

It further said that the impugned regulation must be held to be manifestly arbitrary
and an "unreasonable restriction on the telecom firms' fundamental rights to carry on
business, and is therefore struck down as such."

IN

Consumer may receive compensation for a call drop that may be attributable to the
fault of the consumer himself.
Q Source:
http://indianexpress.com/article/technology/tech-news-technology/trai-regulations-o
n-call-drops-arbitrary-unreasonable-says-sc/

38 How is the International Cricket Council (ICC) Chairman appointed?


A. On the recommendation of the three 'big' cricket boards of India, Australia
and England
B. Elected by the council of ICC
C. Senior-most member of ICC
D. He is generally the Chairman of the largest national cricket board.

(C) Insights Active Learning. | All rights reserved.

www.insightsias.com

24

TEST - 21

User Name :

chandan paswan

Total Marks :

200

Mark Scored :

67.33

User Answer : B
Correct Answer : B
Answer Justification :

IA
S

Justification & Learning: Shashank Manohar, who had twice served as the
President of the Board of Control for Cricket in India (BCCI), was recently elected
unopposed as ICC's (International Cricket Council) first independent Chairman.
ICC appoints umpires and referees that officiate at all sanctioned Test
matches, One Day Internationals (ODIs) and Twenty20 Internationals.
It promulgates the ICC Code of Conduct, which sets professional standards of
discipline for international cricket.
It also co-ordinates action against corruption and match-fixing through its
Anti-Corruption and Security Unit (ACSU).

H
TS

Q
Source:http://www.icc-cricket.com/news/2016/media-releases/94820/shashank-man
ohar-elected-unopposed-as-independent-icc-chairman

IN

SI

39 The Gujral Doctrine, propounded and initiated in 1996, is a milestone in India's foreign
policy. It advocates
A. India should maintain courteous relations with former colonial powers.
B. India should not engage in a war with any nation.
C. India should follow the policy of strategic alignment instead of nonalignment.
D. India should extend unilateral concessions to the smaller neighbours.
User Answer : D
Correct Answer : D
Answer Justification :
Learning: The doctrine is a five-point roadmap to guide the conduct of India's
foreign relations with its immediate neighbours. These five principles are as follows
(for India, the biggest country in South Asia):
With the neighbours like Bangladesh, Bhutan, Maldives, Nepal and Sri
Lanka, India should not ask for reciprocity, but give to them what it can in
good faith.
No South Asian country should allow its territory to be used against the
interest of another country of the region.
No country should interfere in the internal affairs of another country.
All South Asian countries should respect each other's territorial integrity and

(C) Insights Active Learning. | All rights reserved.

www.insightsias.com

25

TEST - 21

User Name :

chandan paswan

Total Marks

200

Mark Scored

67.33

sovereignty.
All South Asian countries should settle all their disputes through peaceful
bilateral negotiations.
Q Source:Chapter 70: Indian Polity: M Laxmikanth

H
TS

IA
S

40 Consider the following with regard to water pollution.


1. Hot water discharged from industries lowers the dissolved oxygen in the water body.
2. Oil spills at sea decrease the oxygen level in the water causing harm to organisms.
Which of the above is/are correct?
A. 1 only
B. 2 only
C. Both 1 and 2
D. None
User Answer : C
Correct Answer : C
Answer Justification :
Justification: Statement 1: The dissolved oxygen in water reduces with increasing
temperature. So, the hotter the water, the lesser is the possibility of its holding
oxygen and aquatic life.

Statement 2: Most of the oil spilled in the ocean does not decompose and remain in
the ocean for years. It uses oxygen as it degrades.

SI

As a result of this, oxygen levels go down. When oxygen levels go down, the
chances of survival of marine animals also goes down.

IN

Q Source: 9th Standard ICSE Textbook: Unit II

41 Consider the following diseases caused due to specific contaminants.


1. Itai Itai: Lead
2. Blue Baby Syndrome: Fluoride
3. Minamata: Mercury
Select the correct answer using the codes below.
A. 1 and 2 only
B. 3 only
C. 2 only
D. 1, 2 and 3
User Answer : B

(C) Insights Active Learning. | All rights reserved.

www.insightsias.com

26

TEST - 21

User Name :

chandan paswan

Total Marks :

200

Mark Scored :

67.33

Correct Answer : B
Answer Justification :
Justification: Statement 1: Water contaminated with Cadmimum causes Itai Itai or
ouch ouch disease which is a painful disease of bones and joints. It can also cause
cancer of lungs and liver.

IA
S

Statement 2: Excess nitrate in water reacts with haemoglobin to form non-functional


methaemoglobin and impairs oxygen transport. It called as blue baby syndrome.
Q Source: 9th Standard ICSE Textbook: Unit II

H
TS

42 Ionizing radiation is an electromagnetic wave with high penetration power. Which of


the following is NOT an ionizing radiation?
A. X-rays
B. Gamma rays
C. Microwaves
D. Cosmic rays
User Answer :
Correct Answer : C
Answer Justification :

Learning: Such radiation carries enough energy to free electrons from atoms or
molecules, thereby ionizing them.

IN

SI

Gamma rays, X-rays, and the higher ultraviolet part of the electromagnetic spectrum
are ionizing, whereas the lower ultraviolet part of the electromagnetic spectrum, and
also the lower part of the spectrum below UV, including visible light (including
nearly all types of laser light), infrared, microwaves, and radio waves are all
considered non-ionizing radiation.
Ionizing radiation can also be generated artificially using X-ray tubes, particle
accelerators, and any of the various methods that produce radioisotopes artificially.
Ionizing radiation is invisible and not directly detectable by human senses
Q Source: 9th Standard ICSE Textbook: Unit II

43 What is the importance of Hukawng valley in Myanmar for India?


A. It is located near the busiest port of Myanmar.

(C) Insights Active Learning. | All rights reserved.

www.insightsias.com

27

TEST - 21

User Name :

chandan paswan

Total Marks

200

Mark Scored

67.33

IA
S

B. It provides an access to the North-eastern region of India.


C. Pulse that is imported in India is mostly grown in this region.
D. It provides India an alternative to the Siliguri Corridor.
User Answer :
Correct Answer : B
Answer Justification :

IN

SI

H
TS

Justification:The image below shows Myanmar and the NE region of India.

It is possible to negotiate the way from the valley to India via a motor vehicle.
Major industry in the valley includes amber and gold mining. It is known for its
tiger reserve.
(We are adding few questions from other sources that may be important. You need
not read these sources completely.)
Q Source: Revision previous tests syllabus: World Geography: Geography by
Surender Singh

(C) Insights Active Learning. | All rights reserved.

www.insightsias.com

28

TEST - 21

User Name :

chandan paswan

Total Marks :

200

Mark Scored :

67.33

IA
S

44 Bioremediation is a waste management technique that involves the use of organisms to


remove or neutralize pollutants from a contaminated site. It can be achieved through
1. Virus
2. Fungi
3. Bacteria
Select the correct answer using the codes below.
A. 1 and 2 only
B. 2 and 3 only
C. 1 and 3 only
D. 1, 2 and 3
User Answer :
Correct Answer : B
Answer Justification :

H
TS

Learning:Statement 2: Mycoremediation is a form of bioremediation in which fungi


are used to decontaminate the area.
Wood-degrading fungi are particularly effective in breaking down aromatic
pollutants (toxic components of petroleum), as well as chlorinated compounds
(certain persistent pesticides).

Statement 3: Bacteria are widely diverse organisms, and thus make excellent players
in biodegradation and bioremediation.

SI

There are few universal toxins to bacteria, so there is likely an organism able to
break down any given substrate, when provided with the right conditions (anaerobic
versus aerobic environment etc).

IN

9th Standard ICSE Textbook: Unit II

45 The Returning Officer of a Parliamentary or assembly constituency is responsible for


the conduct of elections in the Parliamentary or assembly constituency concerned. How is
she appointed?
A. The Election Commission of India (ECI) nominates or designates an officer
of the Government in consultation with the State government
B. The District Magistrate nominates from amongst the officers of the
Constituency in consultation with the ECI
C. The Chief Electoral Officer (CEO) appoints her based on the
recommendation of the District Magistrate
D. The State Government appoints her from amongst the polling officers within
the constituency in consultation with the ECI

(C) Insights Active Learning. | All rights reserved.

www.insightsias.com

29

TEST - 21

User Name :

chandan paswan

Total Marks

200

Mark Scored

67.33

User Answer : A
Correct Answer : A
Answer Justification :
Learning:This is the hierarchy of the election machinery.

IA
S

The Chief Electoral Officer of a state/ Union Territory is authorised to supervise the
election work in the state/Union Territory subject to the overall superintendence,
direction and control of the Election Commission.
Subject to the superintendence, direction and control of the Chief Electoral Officer,
the District Election Officer supervises the election work of a district.
Next lies Returning officer (RO) and Presiding officer.

H
TS

The Presiding Officer with the assistance of polling officers conducts the poll at a
polling station.
The District Election Officer appoints the Presiding Officers and the Polling
Officers.
Q Source:Chapter 65: Indian Polity: M Laxmikanth

IN

SI

46 The term "Primary Deficit" in the Union Budget denotes


A. The fiscal deficit excluding the interest liabilities
B. The effective revenue deficit
C. The Fiscal deficit that is met by non-debt forms of receipts
D. None of the above
User Answer : A
Correct Answer : A
Answer Justification :
Learning: India started using this since the fiscal 1997-98.
It shows the fiscal deficit for the year in which the economy had not to fulfill any
interest payments on the different loans and liabilities which it is obliged to-shown
both in quantitative and percentage of GDP forms.
This is considered a very handy tool in the process of bringing in more transparency
in the government's expenditure pattern.

(C) Insights Active Learning. | All rights reserved.

www.insightsias.com

30

TEST - 21

User Name :

chandan paswan

Total Marks :

200

Mark Scored :

67.33

Any two years for example might be compared and so many things can be found out
clearly such as, which year the government depended more on loans, the reasons
behind higher or lower fiscal deficits, whether the fiscal deficits have gone down
due to falling interest liabilities or some other factors.

IA
S

Q Source: Union Budget 2016-17

H
TS

47 Financial Stability and Development Council is an apex-level body constituted by the


government of India. Its members is/are
1. Governor Reserve Bank of India (RBl)
2. Chief Economic Advisor, Ministry of Finance
3. Secretary, Department of Economic Affairs (DEA)
4. Asia Head, Financial Action Task Force (FATF)
Select the correct answer using the codes below.
A. 1 and 3 only
B. 1, 2 and 3 only
C. 2 and 4 only
D. 1, 2, 3 and 4
User Answer :
Correct Answer : B
Answer Justification :

Learning:It is an autonomous body dealing with macro prudential and financial


regularities in the entire financial sector of India.

SI

The recent global economic meltdown has put pressure on governments and
institutions across the globe to regulate their economic assets. This council is seen as
India's initiative to be better conditioned to prevent such incidents in future.

IN

Chairperson is the Union Finance Minister of India.


Members are Governor Reserve Bank of India (RBl), Finance Secretary and/ or
Secretary, Department of Economic Affairs (DEA), Secretary, Department of
Financial Services (DFS), Chief Economic Advisor, Ministry of Finance, Chairman,
Securities and Exchange Board of India (SEBI), Chairman, Insurance Regulatory
and Development Authority (IRDA), Chairman, Pension Fund Regulatory and
Development Authority (PFRDA), Chairman, Forward Markets Commission
(FMC).
Q
Source:http://www.business-standard.com/article/government-press-release/13th-m

(C) Insights Active Learning. | All rights reserved.

www.insightsias.com

31

TEST - 21

User Name :

chandan paswan

Total Marks

200

Mark Scored

67.33

eeting-of-the-financial-stability-and-development-council-fsdcheld-115110501379_1.html

H
TS

IA
S

48 How is the electoral college of the Vice-President different from that of the President?
1. Vice-President's Electoral College consists of both elected and nominated members
of the Parliament.
2. I Vice-President's Electoral College does not include the members of the state
legislative assemblies.
Which of the above is/are correct?
A. 1 only
B. 2 only
C. Both 1 and 2
D. None
User Answer : A
Correct Answer : C
Answer Justification :
Justification: Statement 1: In the case of President, only elected members
participate.

Statement 2: In the case of President the elected members of the state legislative
assemblies are included.

SI

This is because Vice-President is only a ceremonial head and does not wield any
executive power unless he assumes the office of the President.
Q Source: Chapter on Vice-President: Indian Polity: M laxmikanth

IN

49 Which of the following is the correct order of ecological organization?


A. Biome < Ecosystem < Community < Biosphere
B. Biosphere < Biome < Community < Ecosystem
C. Community < Biome < Biosphere < Ecosystem
D. Community < Ecosystem < Biome < Biosphere
User Answer : D
Correct Answer : D
Answer Justification :
Learning:The image below shows the organization. If unable to access the image,
please refer to the Q source.

(C) Insights Active Learning. | All rights reserved.

www.insightsias.com

32

chandan paswan

Total Marks :

200

Mark Scored :

67.33

H
TS

IA
S

TEST - 21

User Name :

SI

Biomes are defined as the world's major communities, classified according to the
predominant vegetation and characterized by adaptations of organisms to that
particular environment, for e.g. Grassland, Tundra etc.
Q Source:9th Standard ICSE Textbook: Unit I

IN

50 Which of the following are free-living nitrogen-fixing bacteria?


1. Rhizobium
2. Azotobacter
3. Clostridium
Select the correct answer using the codes below.
A. 1 and 2 only
B. 2 and 3 only
C. 1 and 3 only
D. 1, 2 and 3
User Answer :
Correct Answer : B

(C) Insights Active Learning. | All rights reserved.

www.insightsias.com

33

TEST - 21

User Name :

chandan paswan

Total Marks

200

Mark Scored

67.33

Answer Justification :

IA
S

Justification:Two kinds of nitrogen-fixing bacteria are recognized: free-living (nonsymbiotic) bacteria, including the cyanobacteria (or blue-green algae) Anabaena and
Nostoc and genera such as Azotobacter, Beijerinckia, and Clostridium; and
mutualistic (symbiotic) bacteria such as Rhizobium, associated with leguminous
plants (e.g., various members of the pea family), and certain Azospirillum species,
associated with cereal grasses.
So, 1 is not correct.

Rhizobium organisms in the soil recognize and invade the root hairs of their specific
plant host, enter the plant tissues in return for secretions (food) from their host that
encourage their growth and multiplication.

H
TS

Q Source: 9th Standard ICSE Textbook: Unit I

SI

51 The 'Mavalankar rule' in Parliament is related to


A. Rule of Order in a House of Parliament
B. Role of private members in the business of house
C. Conduct of the Parliamentary Committees
D. None of (a), (b) or (c) is correct.
User Answer :
Correct Answer : D
Answer Justification :

IN

Learning: G V Mavalankar, the first Lok Sabha Speaker, had said the main
opposition party's strength must equal the quorum - which is 10 per cent of the total
strength - required for functioning of the House.
As per the Mavalankar rule, any party needs to have at least 10% of the total seats
for its leader to claim the status of the leader of the opposition.
There was a controversy related to this rule in the 16th Lok Sabha as the Main
opposition Congress failed to get more than 55 seats.
Q Source: Chapter on Parliament: Indian Polity: M laxmikanth

52 The Puranas were written to illustrate and expound


A. The rituals followed in ancient India

(C) Insights Active Learning. | All rights reserved.

www.insightsias.com

34

TEST - 21

User Name :

chandan paswan

Total Marks :

200

Mark Scored :

67.33

IA
S

B. The truth of the Vedas


C. The rationale behind the Hindu social structure
D. The history of ancient Kingdoms in India
User Answer : B
Correct Answer : B
Answer Justification :
Learning: The word Purana means 'that which renews the old' and is almost always
mentioned alongwith Itihasa.

H
TS

The fundamental abstruse philosophical and religious truths were expounded


through popular legends or mythological stories.
If something is described as a story, it exerts a greater influence on the human
mind.
Thus, Itihasa combined with narration makes a story seem credible.
Together with the two epics, the Ramayana and the Mahabharata, they are the
origins of many of the stories and anecdotes of the social, religious and
cultural history of India.
Q Source: http://www.ccrtindia.gov.in/literaryarts.php\
"

IN

SI

53 Consider the following statements.Assertion (A): The Rajya Sabha has no power to vote
on the demand for grants.Reason (R): Demand for grants is not even discussed in the Rajya
Sabha.In the context of the above, which of these is correct?
A. A is correct, and R is an appropriate explanation of A.
B. A is correct, but R is not an appropriate explanation of A.
C. A is correct, but R is incorrect.
D. Both A and R are incorrect.
User Answer : B
Correct Answer : C
Answer Justification :
Justification:The Rajya Sabha has no power to vote on the demand for grants; it is
the exclusive privilege of the Lok Sabha.
Moreover, the Rajya Sabha should return the Money bill (or Finance bill or the
Annual budget) to the Lok Sabha within fourteen days.
The Lok Sabha can either accept or reject the recommendations made by Rajya
Sabha in this regard.

(C) Insights Active Learning. | All rights reserved.

www.insightsias.com

35

TEST - 21

User Name :

chandan paswan

Total Marks

200

Mark Scored

67.33

This is because Rajya Sabha is not a directly elected house (like the Lok Sabha) and
the government is not responsible to the Rajya Sabha (like it is to the Lok Sabha).
Q Source:Chapter on Parliament: Indian Polity: M laxmikanth

H
TS

IA
S

54 In Andaman and Nicobar Islands is located strategically at the juncture of the Bay of
Bengal and Andaman Sea. What kind of vegetation can be found there?
1. Tropical Wet Evergreen Forests
2. Moist deciduous forests
3. Mangrove forests
Select the correct answer using the codes below.
A. 1 and 2 only
B. 1 and 3 only
C. 2 and 3 only
D. 1, 2 and 3
User Answer : B
Correct Answer : D
Answer Justification :

Justification & Learning:The Andaman and Nicobar Islands have a tropical


rainforest canopy, made of a mixed flora with elements from Indian, Myanmar,
Malaysian and endemic floral strains.
The Middle Andamans harbours mostly moist deciduous forests.

SI

The North Nicobar Islands are marked by the complete absence of evergreen forests.

IN

While deciduous forests are common in the Andamans, they are almost absent in the
Nicobars.
Q Source: 9th Standard ICSE Textbook: Unit I

55 The Governor cannot make an ordinance without the instructions from the President in
which of the following cases?
1. If the subject of ordinance falls in the concurrent list
2. If the same act of the state legislature would have been invalid without receiving the
President's assent
Which of the above is/are correct?
A. 1 only
B. 2 only

(C) Insights Active Learning. | All rights reserved.

www.insightsias.com

36

TEST - 21

User Name :

chandan paswan

Total Marks :

200

Mark Scored :

67.33

C. Both 1 and 2
D. None
User Answer :
Correct Answer : B
Answer Justification :

IA
S

Justification:Statement 1: Only if a Central law exists on the matter in the


concurrent list, the Governor may require the prior instruction of the President.
Statement 2: Moreover, he had to take President's instructions if a bill containing the
same provisions would have required the previous sanction of the President for its
introduction into the state legislature.

H
TS

Q Source:Chapter on Governor: Indian Polity: M laxmikanth

SI

56 Consider the following statements. Assertion (A): Snow cover on a water body can
cause death of aquatic organisms.Reason (R): Snow cover on a water body can effectively
stop photosynthesis and lead to depletion of vital oxygen in the water body.In the context
of the above, which of these is correct?
A. A is correct, and R is an appropriate explanation of A.
B. A is correct, but R is not an appropriate explanation of A.
C. A is incorrect, but R is correct.
D. Both A and R are incorrect.
User Answer : A
Correct Answer : A
Answer Justification :

IN

Justification: Snow cover of ice on water body can effectively cut off light,
plunging the waters into darkness.
Hence photosynthesis stops but respiration continues. Thus, in shallow lakes,
oxygen gets depleted, and due to lack of oxygen there is large scale death of fishes
and other organisms.
This condition is known as winterkill.
Q Source: 9th Standard ICSE Textbook: Unit I

57 National Science Day is celebrated in order to commemorate


A. The discovery of Raman Effect

(C) Insights Active Learning. | All rights reserved.

www.insightsias.com

37

TEST - 21

User Name :

chandan paswan

Total Marks

200

Mark Scored

67.33

IA
S

B. The finding that plants also have life


C. The first satellite launch from India
D. The first nuclear reactor designed and manufactured in India
User Answer : A
Correct Answer : A
Answer Justification :
Learning: This day marks the epoch-making discovery of Raman Effect by Indian
physicist Chandrasekhara Venkata Raman (CV Raman) on February 28, 1928.
This discovery was awarded with the Nobel Prize in Physics in 1930.

In 2013, the American Chemical Society designated the 'Raman Effect' as an


International Historic Chemical Landmark.

H
TS

The theme of the year 2016 is on "Scientific Issues for Development of the Nation".
Q Source:
http://www.indiatimes.com/news/india/on-national-science-day-here-s-celebrating-o
ne-of-india-s-greatest-minds-physicist-cv-raman-251275.html

IN

SI

58 Which one of the following industries produces the most non-biodegradable waste?
A. Textile Mills
B. Food Processing Units
C. Paper Mills
D. Thermal Power plants
User Answer : D
Correct Answer : D
Answer Justification :
Learning: Thermal power plants mainly produce fly ash.
Integrated iron and steel plants produce blast furnace slag and steel melting slag.
Metallurgical industries such as aluminium, zinc and copper produce anode mud and
tailings.
Fertilizer industries produce gypsum.
Hazardous waste such as inflammables, composite explosives or highly reactive
substance are produced by industries dealing in metals, chemicals, drugs,

(C) Insights Active Learning. | All rights reserved.

www.insightsias.com

38

TEST - 21

User Name :

chandan paswan

Total Marks :

200

Mark Scored :

67.33

pharmaceuticals, dyes, pesticides, rubber goods, etc.


Q Source:UPSC Past year questions: CDS

H
TS

IA
S

59 Aerosol particles may accelerate the process of climate change as


1. They can affect the radiation balance of the earth.
2. They deplete ozone layer.
Which of the above is/are correct?
A. 1 only
B. 2 only
C. Both 1 and 2
D. None
User Answer :
Correct Answer : A
Answer Justification :

Justification:In general, aerosol particles can affect the radiation balance leading to
a cooling or heating effect with
the magnitude and sign of the temperature change largely dependent on aerosol
optical properties, aerosol concentrations, and the albedo of the underlying surface.

A purely scattering aerosol will reflect energy that would normally be absorbed by
the earth-atmosphere system back to space and leads to a cooling effect.

IN

SI

60 The busiest waterway of Europe is formed by Rhine because


A. The river is deep and able to carry large amount of cargo.
B. It has an indented coastline.
C. It passes through one of the most raw material intensive regions of the world.
D. The river passes through one of the most industrialized regions of the world.
User Answer : D
Correct Answer : D
Answer Justification :
Justification: It begins in the Swiss Alps and flows for around 1400 Km through
Germany and the Netherlands to the North Sea.
It forms a natural link between the main industrial region of Germany and the port
of Rotterdam.

(C) Insights Active Learning. | All rights reserved.

www.insightsias.com

39

TEST - 21

User Name :

chandan paswan

Total Marks

200

Mark Scored

67.33

The busiest port of the Europe, Rotterdam, is situated on the mouth of this river.

H
TS

61 Which of the following is/are NOT E-waste?


1. Barium
2. Mercury
3. Brominated Flame Retardants (BFRs)
4. Fluoride
Select the correct answer using the codes below.
A. 2 and 3 only
B. 1 and 4 only
C. 4 only
D. 1, 2 and 3 only
User Answer :
Correct Answer : C
Answer Justification :

IA
S

Q Source: World Geography: 10th NCERT

Justification:Chips and other gold plated components release heavy metals,


brominated flame retardants etc.

Mercury is found in fluorescent tubes, tilt switches (mechanical doorbells,


thermostats) and flat screen monitors.

SI

Brominated Flame Retardants (BFRs) are used as flame retardants in plastics in


most electronics.
Lead, barium and other heavy metals are found in CRTs (TVs).

IN

Q Source:9th Standard ICSE Textbook: Unit II

62 The birth of the novel in India is associated with


A. Bhakti movement during the Medieval era
B. Early Dravidian literature
C. Literature in Pali and Prakrit developed during the early Buddhist age
D. The social reform-oriented movement of the 19th century
User Answer : B
Correct Answer : D
Answer Justification :

(C) Insights Active Learning. | All rights reserved.

www.insightsias.com

40

TEST - 21

User Name :

chandan paswan

Total Marks :

200

Mark Scored :

67.33

Learning:This was a new genre, borrowed from the West, and was characterized by
a spirit of revolt, right from its adoption into the Indian system

IA
S

The first Tamil novel, Pratap Mudaliyar Charitram (1879) by Samuel V. Pillai, the
first Telugu novel, Sri Ranga Raja Charitra (1872) by Krishnamma Chetty, and the
first Malayalam novel, Indu Lekha (1889) by Chandu Menon were written with
didactic intentions and to re-examine evil social customs and practices like
untouchability, caste distinctions, denial of remarriage of widows, etc.
Historical novels were written by Bankim Chandra Chatterjee (Bengali), Hari
Narayan Apte (Marathi), and others, to describe the glorious past of India, and to
instill nationalist fervour in her people.

"

H
TS

Q Source: http://www.ccrtindia.gov.in/literaryarts.php\

IN

SI

63 The "Motion of Thanks", addressed by the President, is put to vote in the first session
after each general election and the first session of every fiscal year. Consider the following
about it.
1. Assertion (A): The motion needs to be passed in Lok Sabha only.
2. Reason (R): The Council of Ministers is mainly responsible to the Lok Sabha.
In the context of the above, which of these is correct?
A. A is correct, and R is an appropriate explanation of A.
B. A is correct, but R is not an appropriate explanation of A.
C. A is incorrect, but R is correct.
D. Both A and R are incorrect.
User Answer : D
Correct Answer : C
Answer Justification :
Justification & Learning: The first session after each general election and the first
session of every fiscal year is addressed by the president. In this address, the
president outlines the policies and programmes of the government in the preceding
year and ensuing year.
This address of the president, which corresponds to the speech from the Throne in
Britain, is discussed in both the Houses of Parliament on a motion called the
Motion of Thanks.
At the end of the discussion, the motion is put to vote. This motion must be passed
in the House. Otherwise, it amounts to the defeat of the government.

(C) Insights Active Learning. | All rights reserved.

www.insightsias.com

41

TEST - 21

User Name :

chandan paswan

Total Marks

200

Mark Scored

67.33

This inaugural speech of the president is an occasion available to the members of


Parliament to raise discussions and debates to examine and criticise the government
and administration for its lapses and failures
Q Source:Chapter on Parliament: Indian Polity: M laxmikanth

H
TS

IA
S

64 Consider the following statements.


1. Assertion (A): To orbit around the Sun, the planet Mars takes lesser time than the
time taken by the Earth.
2. Reason (R): The diameter of the planet Mars is less than that of the Earth.
In the context of the above, which of these is correct?
A. A is correct, and R is an appropriate explanation of A.
B. A is correct, but R is not an appropriate explanation of A.
C. A is incorrect, but R is correct.
D. Both A and R are incorrect.
User Answer : A
Correct Answer : C
Answer Justification :
Justification:Earths diameter is about 12,750 Km, while that of Mars is 6,780 Km.
So, R is correct.

But, orbital period depends on a lot of factors (distance, shape of orbit, mass of
planet etc), and not just the size or weight of the planet.

SI

Mars takes around 1.88 years to revolve around the Sun, so A is incorrect.
Q Source: Revision previous tests syllabus: Geography: 6th NCERT

IN

65 Coral bleaching can occur due to which of the following reason(s)?


1. Elevated sea temperatures
2. Increase in UV radiation in the solar insolation
3. Large oil spills
4. High sediment loading in the ocean
5. Ocean acidification
Select the correct answer using the codes below.
A. 1 and 5 only
B. 2, 3 and 4 only
C. 1, 3 and 5 only
D. 1, 2, 3, 4 and 5

(C) Insights Active Learning. | All rights reserved.

www.insightsias.com

42

TEST - 21

User Name :

chandan paswan

Total Marks :

200

Mark Scored :

67.33

User Answer : C
Correct Answer : D
Answer Justification :

IA
S

Learning: Coral bleaching is a stress response of corals that may be caused by a


number of biotic and abiotic factors, including:

H
TS

Increased (most commonly due to global warming), or reduced water


temperatures
Oxygen starvation caused by an increase in zooplankton levels as a result of
overfishing
Increased solar irradiance (photosynthetically active radiation and ultraviolet
light)
Changes in water chemistry (in particular acidification caused by CO2
pollution)
Increased sedimentation (due to silt runoff)
Bacterial infections; Changes in salinity; Herbicides; Low tide and exposure;
Cyanide fishing
Q Source:9th Standard ICSE Textbook: Unit I

IN

SI

66 The total number of ministers, including the Prime Minister, in the Central Council of
Ministers is
A. Decided by an order of the President
B. Set according to the will of the ruling party
C. Fixed by the Representation of People Act, 1950
D. None of the above is correct.
User Answer : C
Correct Answer : D
Answer Justification :
Justification & Learning: The 91st Amendment Act of 2003 has made the
following provisions to limit the size of Council of Ministers:
The total number of ministers, including the Prime Minister, in the Central Council
of Ministers shall not exceed 15 per cent of the total strength of the Lok Sabha
(Article 75).
The total number of ministers, including the Chief Minister, in the Council of
Ministers in a state shall not exceed 15 per cent of the total strength of the

(C) Insights Active Learning. | All rights reserved.

www.insightsias.com

43

TEST - 21

User Name :

chandan paswan

Total Marks

200

Mark Scored

67.33

Legislative Assembly of that state.


But, the number of ministers, including the Chief Minister, in a state shall not be
less than 12 (Article 164).

IA
S

Q Source:Chapter 67: Indian Polity: M Laxmikanth

H
TS

67 Exoplanets have been in news due to their frequent discoveries. Exoplanets are those
planets which
1. Revolve around the Sun but in distant orbits
2. Have a minor white hole in their proximity
3. Have a Goldilocks zone
Select the correct answer using the codes below.
A. 1 only
B. 1 and 2 only
C. 2 and 3 only
D. None of the above
User Answer : D
Correct Answer : D
Answer Justification :

Justification:Statement 1: It is a planet that does not orbit the Sun and instead orbits
a different star, stellar remnant, or brown dwarf. It is also termed as extrasolar
planet.

IN

SI

Statement 3: It refers to a habitable zone in the planetary system where the


temperature is neither too high nor too low. Such conditions could allow for the
presence on the planets surface of liquid water a key ingredient for life. But, that
need not be present on an exoplanet.
Q
Source:http://www.thehindu.com/sci-tech/science/nasas-official-exoplanet-count-m
ore-than-doubles/article8584605.ece

68 The Union Government has started the process for issuance of Unique Business
Identification Number (UBIN) to
A. Tightly control environmental standards in operating firms
B. Reduce corruption in labour inspection in firms
C. Enable Budding entrepreneurs set up their business without any delays
D. Streamline administration of unlisted functioning small and medium

(C) Insights Active Learning. | All rights reserved.

www.insightsias.com

44

TEST - 21

User Name :

chandan paswan

Total Marks :

200

Mark Scored :

67.33

enterprises
User Answer : C
Correct Answer : C
Answer Justification :

IA
S

Learning: The drive has been launched by the Department of Industrial Policy &
Promotion (DIPP) under the aegis of Union Ministry of Commerce and Industry.
The UBIN would be automatically linked to Ministry Of Corporate Affairs (MCA)
portal after it is issued to the intended beneficiary.
It will be subsequently sent to the CBDT (Central Board of Direct Taxes) which
would nomenclature it as PAN for UBIN.

H
TS

The move is seen as a major step towards boosting ease of doing business and the
start-up India initiative of the government.
Q
Source:http://www.thehindu.com/business/Industry/govt-starts-ubin-drive-for-buddi
ng-entrepreneurs/article8570479.ece

IN

SI

69 The Union Government maintains that currently more than 30 Indian satellites are
operational in the space. Which of the following is/are such satellites?
1. Astrosat
2. Mars Orbiter Mission
3. Antariksha
4. INSAT-3D
Select the correct answer using the codes below.
A. 1 and 2 only
B. 3 and 4 only
C. 1, 2 and 4 only
D. 1, 2, 3 and 4
User Answer :
Correct Answer : C
Answer Justification :
Learning:These include communication Satellites like INSAT-3A, GSAT-6 etc.
Earth Observation Satellites like Resourcesat-2, RISAT-1, Cartosat-1, Oceansat-2,
SARAL, Kalpana-1, Megha-Tropiques and INSAT-3D.

(C) Insights Active Learning. | All rights reserved.

www.insightsias.com

45

TEST - 21

User Name :

chandan paswan

Total Marks

200

Mark Scored

67.33

Navigational Satellites: IRNSS


Space science Satellites: Mars Orbiter Mission (MOM) and Astrosat.

IA
S

About 30 service providers are utilising Ku-band (a part of K band) transponders


onboard indigenous communication satellites for various communication
applications.
These applications include Direct-To-Home Television (DTH), Digital Satellite
News Gathering, telecommunication, VSAT services for banking, tele-education
and business communication etc.
Q Source:http://pib.nic.in/newsite/mbErel.aspx?relid=144875

H
TS

70 The Election Commission of India (ECI) compiles the complete list of members who
are elected in General elections. After the elections are complete, who issues the
notification for the due constitution of the Lok Sabha?
A. President of India
B. Former Council of Ministers
C. Election Commission of India (ECI)
D. Leader of the majority party/coalition
User Answer : C
Correct Answer : C
Answer Justification :

SI

Learning:The ECI issues an appropriate notification for the due constitution of the
House.

IN

With this, the process of elections is complete and the President, in case of the Lok
Sabha, can then convene the house to hold its sessions.
The Governors of the concerned states, in case of State Assemblies, convene the
sessions.
Q Source: Chapter 65: Indian Polity: M Laxmikanth

71 Panchsheel implies the five principles of conduct in international relations. It was


embodied in the Preamble of the Indo-China Treaty on Tibet, signed in 1954. Which of the
following is NOT a part of Panchsheel?
A. Non-interference in each other's internal affairs

(C) Insights Active Learning. | All rights reserved.

www.insightsias.com

46

TEST - 21

User Name :

chandan paswan

Total Marks :

200

Mark Scored :

67.33

IA
S

B. Settlement of boundary disputes in view of historical precedents


C. Non-aggression
D. Peaceful co-existence
User Answer : B
Correct Answer : B
Answer Justification :
Justification & Learning: The five principles were: mutual respect for each others
territorial integrity and sovereignty; non-aggression; non-interference in each others
internal affairs; equality and mutual benefit; and peaceful co-existence.

H
TS

India perceived the Panchsheel as productive of peaceful cooperation of


sovereign nations instead of the balance of terror and the degrading cold war
tensions, being brought about by the rival great power pacts and alliances. India
explained it as based on the concept of universalism as against the concept of the
balance of power.
Panchsheel became very popular and many countries of the world like Burma,
Yugoslavia, Indonesia and so on adopted it.
Panchsheel and non-alignment are the greatest contributions of India to the theory
and practice of international relations

Q Source:Chapter 70: Indian Polity: M Laxmikanth

IN

SI

72 Which of the following themes is NOT likely to be found in early Dravidian Literature?
A. On Good and Evil
B. Heroic Poetry
C. Devotion and divinity
D. All of these themes can be found.
User Answer : A
Correct Answer : D
Answer Justification :
Justification: Early classical Tamil literature is known as Sangam literature
meaning fraternity, indicating mainly two schools of poets, aham (subjective love
poems), and puram (objective, public poetry and heroic).
Aham deals purely with the subjective emotions of the lover, and puram with all
kinds of emotions, mainly the valour and glory of kings, and about good and evil.

(C) Insights Active Learning. | All rights reserved.

www.insightsias.com

47

TEST - 21

User Name :

chandan paswan

Total Marks

200

Mark Scored

67.33

Another striking feature of ancient Tamil literature is the Vaishnava (pertaining to


Vishnu) bhakti (devotional) literature. Effort has been to find out how a man can
achieve divinity.
Q Source: http://www.ccrtindia.gov.in/literaryarts.php\

IA
S

"

H
TS

73 Apart from India, which of the following UN member states have NOT joined the
Treaty on the Non-Proliferation of Nuclear Weapons (NPT)?
1. Israel
2. Nigeria
3. Pakistan
4. South Sudan
5. Sri Lanka
Select the correct answer using the codes below.
A. 1, 3 and 4 only
B. 1, 2, 4 and 5 only
C. 3, 4 and 5 only
D. 1 and 3 only
User Answer :
Correct Answer : A
Answer Justification :

SI

Learning: A total of 191 states have joined the Treaty, though North Korea, which
acceded to the NPT in 1985 but never came into compliance, announced its
withdrawal in 2003.

IN

Four UN member states have never joined the NPT: India, Israel, Pakistan and
South Sudan.
The treaty recognizes five states as nuclear-weapon states: the United States, Russia,
the United Kingdom, France, and China (also the P-5).
India and Pakistan have publicly disclosed their nuclear weapon programs, and
Israel has a long-standing policy of deliberate ambiguity with regards to its nuclear
program.
Q Source:Chapter 70: Indian Polity: M Laxmikanth

74 Temperature inversion refers to the phenomenon where

(C) Insights Active Learning. | All rights reserved.

www.insightsias.com

48

TEST - 21

User Name :

chandan paswan

Total Marks :

200

Mark Scored :

67.33

IA
S

A. Surface of the Earth is warmer than the air above it.


B. Coastal areas witness higher temperatures than usual.
C. Mountain valleys witness orographic rains without convection.
D. None of the above
User Answer : D
Correct Answer : D
Answer Justification :
Justification & Learning:Usually, within the lower atmosphere (the troposphere)
the air near the surface of the Earth is warmer than the air above it, largely because
the atmosphere is heated from below as solar radiation warms the Earth's surface,
which in turn then warms the layer of the atmosphere directly above it.

H
TS

Given the right conditions, the normal vertical temperature gradient is inverted such
that the air is colder near the surface of the Earth. This can occur when, for example,
a warmer, less-dense air mass moves over a cooler, denser air mass.

IN

SI

It can also happen in valleys are the cold air slides down the mountain slope and
warm air moves up.

Q Source:Chapter 14: Goh Cheng Leong: Certificate Physical and Human


Geography

(C) Insights Active Learning. | All rights reserved.

www.insightsias.com

49

TEST - 21

User Name :

chandan paswan

Total Marks

200

Mark Scored

67.33

IA
S

75 Chinook wind refers to


A. Dry winds experienced on the leeward side of mountains
B. Hot winds experienced in the summer in tundra regions
C. Westerly driven winds that bring rainfall to dry areas in the Mediterranean
D. Winds that blow sporadically across Siberia causing massive rainfall in the
interior regions
User Answer : D
Correct Answer : A
Answer Justification :
Learning:The chinook, a native word meaning "snow eater," belongs to a family of
winds experienced in many parts of the world where long mountain chains lie more
or less at right angles to the prevailing wind.

H
TS

It is a warm and dry westerly wind that blows down the Rocky Mountains into the
mountains' eastern slopes and the western prairies.
The Chinook wind provides a welcome respite from the long winter chill.
Q Source: Chapter 14: Goh Cheng Leong: Certificate Physical and Human
Geography

IN

SI

76 As per India's nuclear doctrine (2003), nuclear weapons will only be used in retaliation
against a nuclear attack on Indian territory or on Indian forces anywhere. Nuclear
retaliatory attacks can only be authorised by
A. Cabinet Committee on Security
B. National Security Council
C. Nuclear Command Authority
D. Indian Armed Forces
User Answer : B
Correct Answer : C
Answer Justification :
Learning:The Nuclear Command Authority comprises a Political Council and an
Executive Council. The Political Council is chaired by the Prime Minister. It is the
sole body which can authorise the use of nuclear weapons.
The Executive Council is chaired by the National Security Advisor. It provides
inputs for decision making by the Nuclear Command Authority and executes the
directives given to it by the Political Council.

(C) Insights Active Learning. | All rights reserved.

www.insightsias.com

50

TEST - 21

User Name :

chandan paswan

Total Marks :

200

Mark Scored :

67.33

The Cabinet Committee on Security (CCS) reviews the progress in the


operationalising of Indias nuclear doctrine including the existing command and
control structures, the state of readiness etc.
Q Source:Chapter 70: Indian Polity: M Laxmikanth

H
TS

IA
S

77 Apart from LPG subsidy, which of the following is/are covered by the Direct Benefits
Transfer (DBT) scheme?
1. Student Scholarship
2. Kerosene subsidy
3. Food grains for BPL
Select the correct answer using the codes below.
A. 1 and 2 only
B. 2 and 3 only
C. 1 only
D. 1 and 3 only
User Answer : A
Correct Answer : A
Answer Justification :

Justification:Statement 1: The national scholarship portal initiative is aimed to


facilitate faster and efficient disposal of scholarship applications and timely payment
of the scholarships to the students directly into their bank accounts, through the
process of DBT (Direct Benefit Transfer).

SI

Statement 2:
http://www.business-standard.com/article/economy-policy/govt-announces-direct-be
nefit-transfer-scheme-for-kerosene-subsidy-116010100799_1.html

IN

Statement 3: There was such a proposal and it is still into consideration whether
grain allocation under PDS be replaced by cash memos or account transfers.
Q
Source:http://www.business-standard.com/article/economy-policy/govt-announcesdirect-benefit-transfer-scheme-for-kerosene-subsidy-116010100799_1.html

78 Consider the following statements.


1. Assertion (A): Resolutions approving the Proclamation of Emergency are passed
only by the Lok Sabha.
2. Reason (R): The Rajya Sabha does not represent directly elected representatives.

(C) Insights Active Learning. | All rights reserved.

www.insightsias.com

51

TEST - 21

User Name :

chandan paswan

Total Marks

200

Mark Scored

67.33

IA
S

In the context of the above, which of these is correct?


A. A is correct, and R is an appropriate explanation of A.
B. A is correct, but R is not an appropriate explanation of A.
C. A is incorrect, but R is correct.
D. Both A and R are incorrect.
User Answer : B
Correct Answer : C
Answer Justification :

Justification: The proclamation of Emergency must be approved by both the


Houses of Parliament within one month from the date of its issue.

H
TS

Every resolution approving the proclamation of emergency or its continuance must


be passed by either House of Parliament by a special majority, that is, (a) a majority
of the total membership of that house, and (b) a majority of not less than two-thirds
of the members of that house present and voting.
A proclamation of emergency may be revoked by the President at any time by a
subsequent proclamation. Such a proclamation does not require the parliamentary
approval.

Q Source:Chapter on Parliament: Indian Polity: M Laxmikanth

IN

SI

79 Consider the following matches of Valley with states.


1. Markha Valley : Jammu and Kashmir
2. Dzukou Valley: Himachal Pradesh
3. Sangla Valley: Assam
4. Yumthang Valley: Sikkim
Select the correct answer using the codes below.
A. 3 and 4 only
B. 1 and 2 only
C. 2 and 3 only
D. 1 and 4 only
User Answer :
Correct Answer : D
Answer Justification :
Justification:The Dzukou Valley is a valley located at the border of the Indian
states of Nagaland and Manipur
Sangla valley is in the Kinnaur District of Himachal Pradesh close to the Tibetan

(C) Insights Active Learning. | All rights reserved.

www.insightsias.com

52

TEST - 21

User Name :

chandan paswan

Total Marks :

200

Mark Scored :

67.33

border.
The Yumthang Valley is a grazing pasture surrounded by the Himalayan mountains
in the North Sikkim.

IA
S

Q Source: Past year UPSC questions

H
TS

80 The Self-Employed Women's Association of India (SEWA) is a/an


A. Body formed by the Government of India for the welfare of unorganized
women workers
B. Sub-agency of the United Nations Population Fund (UNPF)
C. Trade Union for poor and self-employed women workers in India.
D. Self-help group registered as a society that provides lending and borrowing
services to any women in India
User Answer :
Correct Answer : C
Answer Justification :
Learning:It was founded in 1972 by the noted Gandhian and civil rights leader Dr
Ela Bhatt.

SI

SEWAs main goals are to organise women workers for full employment. Full
employment means employment whereby workers obtain work security, income
security, food security and social security (at least health care, child care and
shelter). SEWA organises women to ensure that every family obtains full
employment.

IN

It follows the principles of satya (truth), ahimsa (non-violence), sarvadharma


(integrating all faiths, all people) and khadi (propagation of local employment and
self reliance).
Q Source:Often in news

81 Which of the following are set up under an Act of the Parliament?


1. University Grants Commission
2. National Green Tribunal
3. National Consumer Disputes Redressal Commission
Select the correct answer using the codes below.
A. 1 and 2 only
B. 1 and 3 only

(C) Insights Active Learning. | All rights reserved.

www.insightsias.com

53

TEST - 21

User Name :

chandan paswan

Total Marks

200

Mark Scored

67.33

C. 2 and 3 only
D. 1, 2 and 3 only
User Answer :
Correct Answer : D
Answer Justification :

IA
S

Justification: The University Grants Commission (UGC) of India is a statutory


body set up by the Indian Union government in accordance to the UGC Act 1956.
The National Green Tribunal was established under the National Green Tribunal Act
2010 for effective and expeditious disposal of cases relating to environmental
protection and conservation of forests and other natural resources.

H
TS

The National Consumer Disputes Redressal Commission (NCDRC), India is a


quasi-judicial commission in India which was set up in 1988 under the Consumer
Protection Act of 1986.
Q Source:Unit on Statutory bodies: Indian Polity: M Laxmikanth

IN

SI

82 A member of a House belonging to any political party becomes disqualified for being a
member of the House if
1. He is absent from the first meeting of the any new session of the house.
2. He has voluntarily given up the membership of his political party.
3. A no-confidence motion is passed against the individual member.
Select the correct answer using the codes below.
A. 1 and 2 only
B. 2 and 3 only
C. 2 only
D. 3 only
User Answer : C
Correct Answer : C
Answer Justification :
Justification & Learning:Statement 1: There is no such provision, but there is a
provision for disqualification if a member is absent for more than 60 working days
of the house without taking any leave or permission of the speaker.
Statement 2: He will also be disqualified if he votes or abstains from voting in such
House contrary to any direction issued by his political party without obtaining prior
permission of such party and such act has not been condoned by the party within 15
days

(C) Insights Active Learning. | All rights reserved.

www.insightsias.com

54

TEST - 21

User Name :

chandan paswan

Total Marks :

200

Mark Scored :

67.33

Statement 3: No-confidence motion can be passed for the entire council of Ministers
only. It cant be raised for an individual member.
Q Source:Chapter 67: Indian Polity: M Laxmikanth

H
TS

IA
S

83 Use of which of the following was predominant in the Harappan civilization?


1. Painted Gray Ware
2. Northern Black Polished Ware
3. Black and red ware culture
Select the correct answer using the codes below.
A. 1 and 2 only
B. 2 and 3 only
C. 1 and 3 only
D. None of the above
User Answer : C
Correct Answer : D
Answer Justification :

Justification:Statement 1: The Painted Grey Ware culture (PGW) is an Iron Age


culture of the Gangetic plain and the Ghaggar-Hakra valley, lasting from roughly
1200 BCE to 600 BCE. It is contemporary to, and a successor of the Black and red
ware culture.

SI

Statement 2: The Northern Black Polished Ware culture is an urban Iron Age culture
of the Indian Subcontinent, lasting c. 700200 BCE, succeeding the Painted Grey
Ware culture and Black and red ware culture.

IN

It developed beginning around 700 BC, or in the late Vedic period, and peaked from
c. 500300 BC, coinciding with the emergence of 16 great states or mahajanapadas
in Northern India, and the subsequent rise of the Mauryan Empire.
Statement 3: The black and red ware culture (BRW) is an early Iron Age
archaeological culture of the northern Indian subcontinent. It is dated to roughly the
12th 9th century BCE, and associated with the post-Rigvedic Vedic civilization.
Q Source:Revision previous tests syllabus: 7th NCERT: History

84 What will happen to a plant cell if it is kept in a solution having higher water potential?
A. It will shrink in size.
B. It will die due to the disintegration of the cell.

(C) Insights Active Learning. | All rights reserved.

www.insightsias.com

55

TEST - 21

User Name :

chandan paswan

Total Marks

200

Mark Scored

67.33

C. It will swell in size but may not burst.


D. Nothing will happen to the plant cell.
User Answer :
Correct Answer : C
Answer Justification :

IA
S

Justification:A hypotonic solution has higher water potential. It has a lower


osmotic pressure than another solution i.e. has less solute and more water than
another solution

When a plant cell is placed in hypotonic solution, the cell gains water and thus gains
turgor pressure.

H
TS

This leads to the cell getting swollen. But the rigidity of the cell wall, prevents the
cell from bursting.
Q Source: Revision: 9th NCERT: Science

IN

SI

85 Consider the following statements.


1. Assertion (A): Biodiversity hotspots are located only in tropical regions.
2. Reason (R): Regions need to have species richness in order to qualify as
Biodiversity hotspots.
In the context of the above, which of these is correct?
A. A is correct, and R is an appropriate explanation of A.
B. A is correct, but R is not an appropriate explanation of A.
C. A is incorrect, but R is correct.
D. Both A and R are incorrect.
User Answer : C
Correct Answer : C
Answer Justification :
Justification: As you can see in the image below, they extend to non-tropical
regions as well.

(C) Insights Active Learning. | All rights reserved.

www.insightsias.com

56

chandan paswan

Total Marks :

200

Mark Scored :

67.33

H
TS

IA
S

TEST - 21

User Name :

Q Source:ast year UPSC questions

SI

86 The temperature of equatorial regions is moderated due to


A. Cloudiness and heavy precipitation
B. Katabatic winds
C. North-easterly trade winds
D. Cold ocean currents
User Answer : D
Correct Answer : A
Answer Justification :

IN

Justification:Cloudiness and heavy precipitation around 150 250 cm of rainfall or


more in a year, helps to moderate the temperature, so that even at the equator,
climate is not unbearable.
Cloudiness reduces solar insolation, and precipitation lowers the temperature even
from the consistent year long solar insolation that is received at the equator.
Q Source: Chapter 15: Goh Cheng Leong: Certificate Physical and Human
Geography

87 Any question regarding disqualification arising out of defection is to be first decided by


the
A. Supreme Court of India
B. President of India

(C) Insights Active Learning. | All rights reserved.

www.insightsias.com

57

TEST - 21

User Name :

chandan paswan

Total Marks

200

Mark Scored

67.33

C. Election Commission of India


D. Presiding officer of the house
User Answer : D
Correct Answer : D
Answer Justification :

IA
S

Learning: The presiding officer of a House is empowered to make rules to give


effect to the provisions of the Tenth Schedule (provisions for anti-defection).

According to the rules made so, the presiding officer can take up a defection case
only when he receives a complaint from a member of the House. Before taking the
final decision, he must give the member (against whom the complaint has been
made) a chance to submit his explanation.

H
TS

He may also refer the matter to the committee of privileges for inquiry. Hence,
defection has no immediate and automatic effect.
Decision of the Presiding officer is subject to Judicial review.
Q Source:Chapter 67: Indian Polity: M Laxmikanth

IN

SI

88 The North-Western Region mineral belt extends along Aravali in Rajasthan and parts of
Gujarat. Minerals here are associated with Dharwar system of rocks. Which of the
following can be found here?
1. Copper
2. Zinc
3. Uranium
Select the correct answer using the codes below.
A. 1 and 2 only
B. 2 and 3 only
C. 1 and 3 only
D. 1, 2 and 3 only
User Answer :
Correct Answer : D
Answer Justification :
Justification:The Department of Atomic Energy (DAE) has found large deposit of
the mineral in Rohil, Rajasthan.
The deposit is estimated at 5,185 tonnes, which makes it the fourth largest in the
country after Tummalapalle, Chitrial and Peddagattu extension in Andhra Pradesh.

(C) Insights Active Learning. | All rights reserved.

www.insightsias.com

58

TEST - 21

User Name :

chandan paswan

Total Marks :

200

Mark Scored :

67.33

http://www.thehindu.com/news/national/major-uranium-deposit-found-in-rajasthan/
article3836292.ece
Rajasthan is rich in building stones i.e. sandstone, granite, marble. Gypsum and
Fullers earth deposits are also extensive.

IA
S

Dolomite and limestone provide raw materials for cement industry. Gujarat is
known for its petroleum deposits.
Both Gujarat and Rajasthan both have rich sources of salt.
Q Source:Revision: 9th NCERT: Geography

H
TS

89 Miners use canaries to detect the presence of a deadly gas in underground mines which
is
A. Carbon monoxide
B. Methane
C. Hydrogen Sulphide
D. Mine damp gas
User Answer :
Correct Answer : A
Answer Justification :
Learning: Miners collapse and often die even if small quantities of the highly
poisonous CO are present in the air.

SI

When lowered into mines with CO presence, the birds show distress symptoms such
as ruffling of feathers, pronounced chirping and loss of life.

IN

These reactions occur even if 0.15 per cent of CO is present in the air. If the content
is 0.3 per cent the bird shows immediate distress and falls off its perch in two to
three minutes.
A cage of birds is a good indicator in air containing more than 0.15 per cent CO.
Q Source:Revision: 9th NCERT: Geography

90 Consider the following statements.


1. Assertion (A): The Hindu sects of Vaishnavism and Saivism flourished during the
period of Rashtrakutas.
2. Reason (R): Rashtrakutas neglected and even suppressed Budhhism and Jainism.

(C) Insights Active Learning. | All rights reserved.

www.insightsias.com

59

TEST - 21

User Name :

chandan paswan

Total Marks

200

Mark Scored

67.33

IA
S

In the context of the above, which of these is correct?


A. A is correct, and R is an appropriate explanation of A.
B. A is correct, but R is not an appropriate explanation of A.
C. A is correct, but R is incorrect.
D. A is incorrect and R is correct.
User Answer : C
Correct Answer : C
Answer Justification :

Justification:The Rashtrakuta rule was tolerant to multiple popular religions,


Jainism, Vaishnavaism and Shaivism. Buddhism too found support and was popular
in places such as Dambal and Balligavi, although it had declined significantly by
this time.

H
TS

Yet, the growth of all religions did not affect the progress of Jainism under the
patronage of Rashtrakuta kings and officers. Almost one third of the population of
the Deccan were Jains.
There were some prosperous Buddhist settlements at places like Kanheri, Sholapur
and Dharwar.

Q Source:Revision past tests syllabus: 11th TamilNadu History Textbook

IN

SI

91 Casual employment implies


A. Self-employment
B. Employment in the unorganized sector
C. Disguised unemployment
D. None of the above
User Answer : B
Correct Answer : D
Answer Justification :
Justification: Unorganised sector, also known as informal sector or own account
enterprises, refers to all unlicensed, self-employed or unregistered economic activity
such as owner manned general stores, handicrafts and handloom workers, rural
traders, farmers, etc. It does not necessarily involve casual employment. So, (b) is
not the correct answer.
It is when a labour is hired without a contract, paid irregular wages, avails no social
security and can be fired at the will of the employer, it is termed as casual
employment.

(C) Insights Active Learning. | All rights reserved.

www.insightsias.com

60

TEST - 21

User Name :

chandan paswan

Total Marks :

200

Mark Scored :

67.33

Q Source:Revision: 11th NCERT: Indian Economic Development

IA
S

92 Confidence Building Measures (CBMs) that are often discussed in context of IndoChina negotiations relate to
A. People to people contact across the borders
B. Increase in travel facilities between the countries
C. Joint initiative to tackle socio-economic problems
D. Information exchange among the armed forces of both nations
User Answer :
Correct Answer : D
Answer Justification :

H
TS

Justification:CBM are actions taken to reduce fear of attack by both (or more)
parties in a situation of tension with or without physical conflict.
For e.g. The Indo-China agreement provides for border security and confidence
measures between the parties to the agreement.
It calls for military disclosure when the parties are undertaking border exercises and
for the reduction of troop levels in the border areas.

It also allows the parties to observe and inspect troop movements in each other
territory upon invitation.

SI

Q Source: Often in news due to Indo-China border incidents

IN

93 Consider the following about Narmada River.


1. It is the longest west flowing river in India.
2. It flows in a linear rift valley.
3. No protected area lies within the river-basin of Narmada.
Select the correct answer using the codes below.
A. 1 and 2 only
B. 2 and 3 only
C. 1 and 3 only
D. 1, 2 and 3 only
User Answer : A
Correct Answer : A
Answer Justification :
Justification:Statement 1 and 2: It is one of only three major rivers in peninsular

(C) Insights Active Learning. | All rights reserved.

www.insightsias.com

61

TEST - 21

User Name :

chandan paswan

Total Marks

200

Mark Scored

67.33

India that run from east to west (longest west flowing river), along with the Tapti
River and the Mahi River.
It is one of the rivers in India that flows in a rift valley, flowing west between the
Satpura and Vindhya ranges.

IA
S

The other rivers which flow through rift valley include Damodar River in Chota
Nagpur Plateau and Tapti. The Tapti River and Mahi River also flow through rift
valleys, but between different ranges
Statement 3: Satpura National Park, Mandla Plant Fossils National Park, Dindori
National fossils park etc lie in the river basin of Narmada.

H
TS

Q Source: 9th NCERT Geography

SI

94 What is the principal difference between the average chemistry of the crust and the
average chemistry of Earth as a whole?
A. The crust is relatively enriched in less dense compounds and relatively
depleted in iron.
B. The crust is relatively enriched in magnesium and relatively depleted in
oxygen.
C. Earth as a whole has a greater abundance of silicon than does the crust.
D. The crust contains a greater abundance of heavier elements than does Earth as
a whole.
User Answer :
Correct Answer : A
Answer Justification :

IN

Justification:The most common chemical elements in the crust are oxygen (46.6%),
silicon (27.7), aluminium (8.1), iron (5.0), calcium (3.6), potassium (2.8), sodium
(2.6), and magnesium (2.1).
Typical mantle rocks have a higher magnesium to iron ratio and a smaller proportion
of silicon and aluminium than the crust.
The solid, inner core of earth is iron and has a radius of about 760 miles. It is
surrounded by a liquid, outer core composed of a nickel-iron alloy.
Q Source: Revision Previous Tests syllabus: 11th NCERT: Fundamentals of
Physical Geography

(C) Insights Active Learning. | All rights reserved.

www.insightsias.com

62

TEST - 21

User Name :

chandan paswan

Total Marks :

200

Mark Scored :

67.33

IA
S

95 The equatorial environment is best suited to plantation agriculture due to the presence of
A. Good rainfall, humid climate, cheap labour and good markets in Europe and
North America
B. Hot climate and absence of disease causing pests that reduce plantation yield
C. Acidic soil and dry weather for most of season that supports the growth of
plantation crops especially dry fruits
D. Sandy soil, hot weather and large markets in temperate regions
User Answer : A
Correct Answer : A
Answer Justification :

H
TS

Learning: Plantation agriculture is confined within tropical areas, i.e., both sides of
the equator. Plantations exist on every continent possessing a tropical climate. The
plantation system however is considerably older in tropical America than in Asia
and Africa.
Plantation agriculture is an export-oriented agriculture.
Plantation agriculture is the product of colonialism. Plantations have been developed
in response to a demand in Europe for foods, spices, fibers, and beverages, which
because of climatic constraints, could be produced only in the tropics or sub-tropics.

Q Source:Chapter 15: Goh Cheng Leong: Certificate Physical and Human


Geography

IN

SI

96 Ecotone is a junction between two or more diverse ecosystems. Which of the following
is/are the characteristics of Ecotones?
1. A well developed ecotone may contain organisms that are entirely different from the
adjoining communities.
2. Ecotones may display edge effect where the population density of some species is
more in ecotone than adjoining communities.
Which of the above is/are correct?
A. 1 only
B. 2 only
C. Both 1 and 2
D. None
User Answer :
Correct Answer : C
Answer Justification :
Justification:Statement 1: Because an ecotone is the zone in which two

(C) Insights Active Learning. | All rights reserved.

www.insightsias.com

63

TEST - 21

User Name :

chandan paswan

Total Marks

200

Mark Scored

67.33

communities integrate, many different forms of life have to live together and
compete for space. Therefore, an ecotone can create a diverse ecosystem.
So, the ecotone contains not only species common to the communities on both sides;
it may also include a number of highly adaptable species that tend to colonize such
transitional areas

Q Source: 9th Standard ICSE Textbook: Unit I

IA
S

Statement 2: If different species can survive in both communities of the two biomes,
then the ecotone is considered to have species richness that may be greater than
adjoining communities.

SI

H
TS

97 Consider the following statements.


1. Assertion (A): There cannot be quorum in Parliament if the Leader of Opposition
(LoP) is absent from House proceedings or is not selected by the opposition.
2. Reason (R): All censure motions or resolutions against the government on behalf of
the house can be raised only by the Leader of Opposition (LoP).
In the context of the above, which of these is correct?
A. A is correct, and R is an appropriate explanation of A.
B. A is correct, but R is not an appropriate explanation of A.
C. A is correct, but R is incorrect.
D. Both A and R are incorrect.
User Answer : D
Correct Answer : D
Answer Justification :

IN

Justification:Quorum in Parliament simply means presence of more than 10% of


the actual strength of the House.
It is not related to the absence of presence of any particular member. If LoP or even
the Leader of the house (lets say PM) is absent, the house still conducts business.
Censure motions or resolutions against the government on behalf of the house can
be raised by any opposition member. So, R is also wrong.
Q Source: Chapter on Parliament: Indian Polity: M laxmikanth

98 The policy of abolition of landlordism and of "land belonging to the tiller" was adopted
by the Indian National Congress (INC) since

(C) Insights Active Learning. | All rights reserved.

www.insightsias.com

64

TEST - 21

User Name :

chandan paswan

Total Marks :

200

Mark Scored :

67.33

IA
S

A. It was constituted in 1885.


B. The beginning of Non-cooperation movement
C. Karachi Session 1931
D. A 1945 resolution by the Congress Working Committee
User Answer : B
Correct Answer : D
Answer Justification :
Learning: In 1945, the Congress Working Committee adopted a resolution for the
abolition of landlordism and the granting of land to the tiller.
This was also the time for the consolidation of the Indian capitalist class within the
national movement.

H
TS

This was not adopted earlier as INC felt it may generate backlash with the landlord
community and may not augur well for the Indian national movement.
Q Source: Revision: Indias struggle for Independence: Bipin Chandra

IN

SI

99 For Gandhiji, non-violence was a matter of principle. But for other leader it was a
matter of policy in the independence struggle. It was adopted because
1. Non-violence would broaden the base of the national movement.
2. British never followed the rule of law.
3. Unarmed opposition was never suppressed by the British.
Select the correct answer using the codes below.
A. 1 and 2 only
B. 2 and 3 only
C. 1 only
D. 1 and 3 only
User Answer : D
Correct Answer : C
Answer Justification :
Justification:It was essential to achieve these objectives:

To broaden the participation in the movement.


To fight British without arms. They respected rule of law and generally did
not crush movements which did not violate the law. It put the rulers in a
dilemma. By not acting their position would weaken. By acting their image of
a benevolent ruler would be eroded.

(C) Insights Active Learning. | All rights reserved.

www.insightsias.com

65

TEST - 21

User Name :

chandan paswan

Total Marks

200

Mark Scored

67.33

Women were able to participate in this way.


Disarmed people hardly had any alternative.
Q Source: Revision: Indias struggle for Independence: Bipin Chandra

H
TS

IA
S

100 Consider the following statements.


1. Assertion (A): The pyramid of energy in an ecosystem is always upright.
2. Reason (R): There is loss of energy at each trophic level in being transferred to the
higher trophic level.
In the context of the above, which of these is correct?
A. A is correct, and R is an appropriate explanation of A.
B. A is correct, but R is not an appropriate explanation of A.
C. A is correct, but R is incorrect.
D. Both A and R are incorrect.
User Answer : A
Correct Answer : A
Answer Justification :
Justification & Learning: Pyramid of energy is a graphic representation of the
amount of energy trapped per unit time and area in different trophic level of a food
chain with producers forming the base and the top carnivores at the tip.

SI

Pyramid of energy is always upright. It is so because at each transfer about 80 - 90%


of the energy available at lower trophic level is used up to overcome its entropy and
to perform metabolic activities. Only about 10% of the energy is available to next
trophic level.

IN

Q Source:9th Standard ICSE Textbook: Unit I

(C) Insights Active Learning. | All rights reserved.

www.insightsias.com

66

You might also like